Last min SBA 2 Flashcards
A systematic review on the use of apixaban for left ventricular thrombus has shown a significant reduction in hospitalisation and mortality. Twenty randomised and non-randomised studies were included in the meta-analysis. The authors were concerned about the potential impact of studies with significant results being published more frequently than studies with negative findings and decided to investigate for publication bias.
What is the most appropriate method of assessing for publication bias in this study?
Forest plot
Funnel plot
GRADE analysis
Meta-regression analysis
Sensitivity analysis
Funnel plot for publication bias
A 64-year-old woman presents to her GP with complaints of diffuse joint pain, which has been particularly severe in her right first interphalangeal joint and left wrist for the past six months. The pain is worse in the morning and improves as the day passes. She reports having had a sore throat one week before the onset of her symptoms.
On examination, swelling of the distal interphalangeal joints on her left hand is observed, and the whole fingers seem swollen.
What is the most likely diagnosis?
Osteoarthritis
Psoriatic arthritis
Reactive arthritis
Rheumatoid arthritis
Systemic lupus erythematosus
Psoriatic arthritis
Inflammatory arthritis involving DIP swelling and dactylitis points to a diagnosis of psoriatic arthritis
A 40-year-old woman presents to the rheumatology clinic with a 3-month history of myalgia and widespread bony tenderness. She describes increased fatigue and weakness whilst lifting heavy objects. Her past medical history includes coeliac disease.
On examination, there is tenderness over the shoulder girdle and arms. There is no associated joint stiffness. She has a waddling gait.
Blood tests are taken:
Calcium 1.9 mmol/L (2.1 - 2.6)
Phosphate 0.8 mmol/L (0.8 - 1.4)
ALP 176 u/L (30 - 100)
What is the most likely diagnosis?
Myositis
Myotonic dystrophy
Osteomalacia
Osteoporosis
Polymyalgia rheumatica
Osteomalacia
dermatomyositis related to
malignancy
A 58-year-old undergoes a triple assessment after finding a lump in the right upper lateral quadrant of her breast. Her last menstrual period was 8 years ago, she has never used any hormonal contraceptives or hormone replacement therapy and has no other past medical history.
A biopsy shows the presence of ductal carcinoma in situ that is progesterone receptor-negative, HER2-negative, and oestrogen receptor-positive. She is offered a lumpectomy with adjuvant radiotherapy and endocrine therapy.
What is the mechanism of action of the most likely drug she will be given?
Complete oestrogen receptor antagonism
GnRH receptor agonism
GnRH receptor antagonism
Inhibition of peripheral oestrogen synthesis
Partial oestrogen receptor antagonism
Inhibition of peripheral oestrogen synthesis- aromatase inhibitors
A 48-year-old man is due to undergo a laparotomy for small bowel obstruction.
The correct answer is: Endotracheal intubation
A 63-year-old man has been on the intensive care unit for a week with adult respiratory distress syndrome complicating acute pancreatitis. He has required ventilation and is still being mechanically ventilated.
Tracheostomy 33%
Tracheostomy is often used to facilitate long term weaning. The percutaneous devices are popular. These involve a seldinger type insertion of the tube. A second operator inserts a bronchoscope to ensure the device is not advanced through the posterior wall of the trachea. Complications include damage to adjacent structures and bleeding (contra indication in coagulopathy).
A 47-year-old man is brought to the emergency department with a laceration on his left arm after falling at home. There was copious blood at the scene and the patient received a transfusion of one unit of packed red blood cells on arrival at the hospital.
The patient remains haemodynamically stable throughout and his wound is numbed with 10mls of 2% lidocaine prior to suturing by a junior member of the team.
Before suturing can begin the patient becomes profoundly tachycardia and hypotensive with a heart rate of 150 bpm and blood pressure of 70/40mmHg.
What is the most appropriate reversal agent?
IM glucagon
IV atropine
IV labetalol
IV lipid emulsion
IV magnesium sulfate
IV lipid emulsion
which anaesthetic can cause adrenal suppression
Etomidate
A 9-year-old boy is diagnosed as having Attention Deficit Hyperactivity Disorder and started on methylphenidate. Which one of the following should be monitored during treatment?
Visual acuity
Electrocardiogram to measure QRS duration every 12 months
Growth
Urinalysis
Liver function tests
Growth